Beban terpusat P akan diterapkan pada balok dengan bahan granit sesuai dengan gambar diatas. Jika diketahui nilai maksimum tegangan geser pada balok tersebut adalah sbesar 2,5 ksi maka tentukan nilai (a) beban terpusat P, (b) bidang permukaan yang memiliki tegangan geser maksimal, (c) Nilai maksimum tegangan normal pada balok. \begin{aligned}A_o&=&s\ x\ s\\ A_o&=&6\ x\ 6\\ A_o&=&36\ {in}^2\\ \end{aligned} Guru Solution (a) \begin{aligned}\tau_{maximum}&=&2,5\ ksi\\ \tau_{maximum}&=&\frac{\left|P\right|}{{2A}_o}\\ \left|P\right|&=&{2A}_{o\bullet}\tau_{maximum}\\ \left|P\right|&=&\left(2\right)\left(36\right)\left(2,5\right)\\ \left|P\right|&=&180\ kips\\ \end{aligned}Guru Solution (b) \begin{aligned} \theta&=&45°\\ 2\theta&=&2\bullet45°\\&=&90°\\ Sin2\theta&=&Sin2\bullet45°\\ Sin90°&=&1 \end{aligned} Guru Solution (c) \begin{aligned}\sigma_{45°}&=&\frac{P}{A_o}{cos}^2{45}^o\\ \sigma_{45°}&=&\frac{P}{A_o}\frac{1}{2}\\ \sigma_{45°}&=&\frac{-180}{\left(36\right)\left(2\right)}\\ \sigma_{45°}&=&-2.5\ ksi \end{aligned}Guru Solution (d)\begin{aligned}\sigma_{max}&=&\frac{P}{A_o}\\ \sigma_{max}&=&\frac{-180}{36}\\ \sigma_{max}&=&-5\ ksi\end{aligned}
Tidak ada komentar:
Posting Komentar